CISI Revision

You might also like

Download as docx, pdf, or txt
Download as docx, pdf, or txt
You are on page 1of 98

Chapter 4 The FCA’s Conduct of Business and Client Assets Sourcebooks

1Where the Conduct of Business Sourcebook requires information to be


provided to a client, the firm is permitted to provide this by means of a website
subject to which one of the following conditions?

The client must specifically agree to this approach


The client must be a market counterparty
The firm must hold exempt status under the Data Protection Act
The firm must be a non-MiFID firm
Reference chapter: Chapter 4 Section 1.5

Answer Explanation
With specific reference to website conditions, the FCA requires that a number
of conditions are met, including the fact that the client must specifically consent
to having information provided to them in that form.

2The COBS rules do not apply to which type of organisation?

A UK MiFID firm providing services for UK clients in the UK


A UK MiFID firm providing services from a branch in France to a client in
Germany
An EEA MiFID firm providing services to UK customers from a branch in the UK
An EEA MiFID firm providing services to UK customers from its home office in
France
Reference chapter: Chapter 4, Section 1.2

Answer Explanation
The rules in COBS apply on a geographic basis so that firms are subject to
COBS if they carry on any of a range of activities from an establishment
maintained by them or their appointed representative in the UK. An EEA MiFID
firm providing services from its home office would be subject to Home State
conduct of business rules.
3Which one of the following rules can be disapplied for firms carrying on ‘eligible
counterparty business’?

Accepting deposits
Designated investment business
Reporting requirements to clients
Dealing in long-term life policies
Reference chapter: Chapter 4, Section 1.3

Answer Explanation
A range of COBS rules are disapplied, in certain cases, for firms carrying on
‘eligible counterparty business’. These include COBS 16, which covers
reporting requirements to clients.

4Where firms accept deposits, this activity is:

Considered an ancillary service for regulatory purposes


Covered by the Conduct of Business Sourcebook rules
Treated as market counterparty business for MiFID purposes
Excluded from the Financial Services and Markets Act 2000
Reference chapter: Chapter 4, Section 1.3

Answer Explanation
The activities that are covered by COBS rules are: accepting deposits;
designated investment business; long-term insurance business in relation to life
policies; and activities relating to the three previously named activities.

5Which of the following activities is partially exempt from COBS?

Managing designated investments


Dealing in investments as principal
Dealing in designated investments as agent
Carrying on eligible counterparty business
Reference chapter: Chapter 4, Section 1.3

Answer Explanation
Designated investment businesses are subject to COBS. A range of COBS
rules are disapplied, in certain cases, for firms carrying on eligible counterparty
business.

6If ABC are appointed representatives of firm XYZ, then:

ABC’s clients will all be classed as XYZ’s market counterparty clients


ABC must process all transactions on an execution-only basis
XYZ is responsible for the compliance of ABC’s financial promotions
XYZ is not permitted to have any other appointed representatives
Reference chapter: Chapter 4, Section 1.4

Answer Explanation
The COBS rules apply to firms in relation to the relevant activities carried on for
them by their appointed representatives. In particular, firms must ensure that
they comply with the COBS rules when they communicate financial promotions
via their appointed representatives.

7An appointed representative of an insurance company wishes to make a


financial promotion. Who is responsible for its content?

Another authorised firm with specific permission to approve such promotions


The appointed representative
Approval is not required as it is exempt
The authorised insurance company
Reference chapter: Chapter 4, Section 1.4

Answer Explanation
Technically, a financial promotion communicated by an appointed
representative is an exempt promotion and so does not need approval but the
authorised firm is responsible for the content and any actions of their appointed
representatives.

8Electronic business is considered by the FCA to be a ‘durable’ medium if it:

Enables the recipient to store the information


Requires a verbal response
Is via a non-interactive website
Contains a shelf life message at the start
Reference chapter: Chapter 4, Section 1.5

Answer Explanation
Where the rules refer to information being transmitted or provided in a ‘durable
medium’, this means: paper; or any instrument which lets the recipient store the
information so that they can access it for future reference, for an appropriate
time and on an unchanged basis. It includes storage on a PC but excludes
internet sites, unless they meet the requirement for storage and retrieval.

9The rules on the recording of telephone and electronic communications were


introduced to address what type of activity?

Appropriateness
Conflicts of interest
Market abuse
Suitability
Reference chapter: Chapter 4, Section 1.6

Answer Explanation
Preventing, detecting and deterring market abuse is the reason why the
regulator developed rules requiring certain firms to record and retain telephone
conversations and other electronic communications.
10Which category of client has the highest level of protection under the Conduct
of Business rules?

Elective eligible counterparties


Elective professional clients
Per se professional clients
Retail clients
Reference chapter: Chapter 4, Section 2.1.3

Answer Explanation
Retail clients are afforded the highest protections under the COBS rules.

11For a large undertaking to be treated as a per se professional client for MiFID


business, two conditions must be met. The company must have:

A balance sheet of £12.5m, net turnover of £25m, and own funds of £2m
A balance sheet of £20m, net turnover of £40m, and own funds of £2m
A balance sheet of €12.5m, net turnover of €25m, and own funds of €2m
A balance sheet of €20m, net turnover of €40m, and own funds of €2m
Reference chapter: Chapter 4, Section 2.1.3

Answer Explanation
Large undertakings can be classified as a per se professional client for MiFID
business where they meet any two of the following size requirements on a
company basis: a balance sheet total of €20 million, a net turnover of €40 million,
or own funds of €2 million.

12Which one of the following is one of the criteria for the quantitative test when
categorising elective professional clients for MiFID business?

The investor has previously carried out 15 significantly sized transactions


The investor has a financial portfolio exceeding €500,000
The investor has requested this categorisation based on their own knowledge
and experience
The investor is acting as an agent for an elective professional client
Reference chapter: Chapter 4, Section 2.1.5

Answer Explanation
For MiFID business, a client may be treated as an elective professional client if
they meet two of the three criteria for the quantitative test: an average of ten
significantly sized transactions have been carried out on the relevant market in
each of the past four quarters; the size of the client’s financial portfolio exceeds
€500,000; and the investor works or has worked as a professional in the
financial services sector for at least a year on a basis which would require
knowledge of the transactions envisaged.

13As part of the client status categorisation process, a client successfully


satisfied the qualitative and quantitative tests. Which client status has
consequently been used?

Retail client
Per se professional client
Elective professional client
Eligible counterparty
Reference chapter: Chapter 4, Section 2.1.5

Answer Explanation
When a client successfully satisfies both the qualitative and quantitative tests
as part of the client status categorisation process, they may be treated as an
'elective professional client'.

14Where appropriate, under the quantitative test for categorising elective


professional clients, at least how many significantly sized transactions are
required to have been carried out on average in each of the past four quarters?

10
15
20
25
Reference chapter: Chapter 4, Section 2.1.5

Answer Explanation
One of the three criteria for the quantitative test is that the client has carried out,
on average, ten significantly sized transactions on the relevant market in each
of the past four quarters.

15Where a firm conducts designated investment business with a retail client,


when is the latest that a client agreement should normally be provided?

Before the relevant service is provided


Immediately after the relevant service is provided
7 days after the relevant service is provided
14 days after the relevant service is provided
Reference chapter: Chapter 4, Section 2.2

Answer Explanation
The client agreement must be provided in good time before a retail client is
bound by any agreement relating to designated investment business.

16A client agreement that covers pension transfer business should be retained
for how long?

Three years from the date of signing


Five years from the date of transfer
Five years from the end of the relationship
Indefinitely
Reference chapter: Chapter 4, Section 2.2

Answer Explanation
Client agreements for pension transfer, pension opt-outs or free-standing
additional voluntary contributions must be retained indefinitely. The rationale
behind the rule is that it could be many years before any problems with the
advice come to light given the long-term nature of pension products.

17A firm must provide a client agreement prior to conducing MiFID business for
which of the following types of client?

Retail clients only


Professional clients only
Eligible counterparties and professional clients only
Professional clients and retail Clients only
Reference chapter: Chapter 4, Section 2.2

Answer Explanation
The requirement for a client agreement applies to designated investment
business carried on for a retail client and in relation to MiFID or equivalent third-
country business, a professional client. It also applies to ancillary services for
MiFID business or equivalent third-country business.

18Disclosure of information to clients about safeguarding investments relates


to what major set of FCA rules?

Best execution
CASS
Conflicts of interest
Periodic reporting
Reference chapter: Chapter 4, Section 2.3.4

Answer Explanation
The rules on the provision of information concerning safeguarding of
designated investments belonging to clients relate to the extensive rules on the
safeguarding of client assets.
19The rules on financial promotions apply to firms communicating or approving
a financial promotion other than:

A promotion for a non-investment insurance contract


A promotion for an authorised collective investment scheme
A stakeholder pension
An endowment policy
Reference chapter: Chapter 4, Section 3.1

Answer Explanation
The rules apply to firms communicating with clients regarding their designated
investment business and communicating or approving a financial promotion
other than - for qualifying credit, a home purchase plan or a home reversion
plan, a promotion for a non-investment insurance contract, or the promotion of
an unregulated CIS which it is not permitted to approve.

20A financial promotion is issued by an FCA-authorised firm and includes a


reference to the FCA but also refers to matters not regulated by the FCA. Which
one of the following is most accurate?

The promotion is not allowed


The promotion is allowed, but no mention should be made of the FCA
The promotion must not be communicated to retail clients
The promotion must make it clear which matters are not regulated by the FCA
Reference chapter: Chapter 4, Section 3.1.1

Answer Explanation
In connection with communications which are financial promotions, firms that
mention the FCA must ensure that matters that are not regulated by the FCA
are clearly stated as such.

21Which one of the following is a requirement of Principle 7 in relation to


financial promotions?
The firm discloses all charges and commissions relating to the promotion
The firm pays due regard to the customer’s interests
The information communicated is clear, fair and not misleading
The investment must be suitable for the customer
Reference chapter: Chapter 4, Section 3.1.1

Answer Explanation
Principle 7 states that a firm must pay due regard to the information needs of
its clients and communicate information to them in a way that is clear, fair and
not misleading.

22The rules on financial promotions are particularly designed to ensure


consistency with which two of the Principles for Businesses?

Principle 2 and Principle 6


Principle 3 and Principle 4
Principle 6 and Principle 7
Principle 7 and Principle 10
Reference chapter: Chapter 4, Section 3.1.1

Answer Explanation
The financial promotion rules are consistent with Principles 6 and 7 of the
Principles for Businesses - Principle 6 – a firm must pay due regard to the
interests of its customers and treat them fairly, Principle 7 – a firm must pay
due regard to the information needs of its clients and communicate information
to them in a way which is clear, fair and not misleading.

23FCA rules on financial promotions specifically reinforce which one of the


following Principles for Businesses?

Principle 5 – Market conduct


Principle 6 – Customers' interests
Principle 8 – Conflicts of interest
Principle 9 – Customers: relationships of trust
Reference chapter: Chapter 4, Section 3.1.1

Answer Explanation
Principle 5 is concerned with proper standards of market conduct; Principle 8 is
concerned with managing conflicts fairly; and Principle 9 requires that advice is
suitable to the customer's needs. Principles 6 and 7 are specifically reinforced
by FCA rules on financial promotions.

24Which general Conduct of Business rule states that communication must be


appropriate to the level of knowledge of the customer?

Suitability
Clear, fair and not misleading
Inducements
Know your customer
Reference chapter: Chapter 4, Section 3.1.1

Answer Explanation
Firms must ensure that communications relating to designated investment
business are fair, clear and not misleading. The way in which this is achieved
should be appropriate and proportionate and take account of the means of
communication and what information the communication is intended to convey.
So, for example, communications aimed at professional clients may not need
to include all the same information as those aimed at retail clients.

25Under FCA rules all financial promotions to a retail client are obliged to
contain which one of the following items?

Statement requesting future contact


The firm’s name
Comparisons with different types of investments
Past performance information
Reference chapter: Chapter 4, Section 3.5
Answer Explanation
Firms must ensure that financial promotions to retail clients adhere to certain
rules. One of these is that the firm’s name must be included on the
communication.

26Where a financial promotion includes past performance figures in relation to


a fund established 15 years ago, the figures must include performance over
what period?

The previous 12 months


The previous 24 months
The previous 3 years
The previous 5 years
Reference chapter: Chapter 4, Section 3.6.1

Answer Explanation
Firms must ensure that information, including indications of past performance,
is such that it covers at least the immediately preceding five years (or the whole
period that the investment has been offered/the financial index has been
established/the service has been provided (as applicable) if this is less than five
years).

27Where firms include past performance data on the products they promote,
this must be:

Not the most prominent element of the financial promotion


Of equal prominence to other elements of the financial promotion
Past performance data may not be used in a financial promotion
The most prominent element of the financial promotion
Reference chapter: Chapter 4, Section 3.6.1

Answer Explanation
Firms must ensure that information including indications of past performance is
such that the past performance indication is not the most prominent feature.

28The presentation of simulated past performance in a financial promotion


must meet what requirement?

It can be used to project future performance


It may only be based on the performance of an index
It may only use agreed growth rates set by the regulator
It must be based on the performance of similar investments
Reference chapter: Chapter 4, Section 3.6.2

Answer Explanation
When firms give figures based on simulated past performance it must ensure
that they: relate to an investment or financial index; are based on the actual
past performance of one or more investments/indices which are the same as,
or underlie, the investments being simulated; meet the rules for past
performance; and contain a prominent warning that they relate to simulated
past performance and that past performance is not a reliable indicator of future
performance.

29Cold calling may be made to retail clients for which one of the following
products?

OEIC ISAs
Unregulated collective investment schemes
Financial futures
Warrants
Reference chapter: Chapter 4, Section 3.8.2

Answer Explanation
Cold calls can be made to retail clients where there is an existing relationship
and the client would envisage receiving such a call; the call relates to a
generally marketable packaged product which is neither a higher volatility fund
nor a life policy linked to such a fund; or it relates to a ‘controlled activity’ relating
to a limited range of investments, including deposits and readily realisable
investments other than warrants or generally marketable non-geared packaged
products.

30Undertaking more trading than necessary in order to generate commission


is known as?

Churning
Front running
Layering
Switching
Reference chapter: Chapter 4, Section 4.1.1

Answer Explanation
Churning is the activity of over-dealing/trading more frequently for a client in
order to generate additional fees/commissions for the firm.

31What primary factor should a firm take into account when determining how
frequently to deal for a client?

The client’s country of origin


The client’s investment strategy
The client’s level of knowledge
The client’s tax position
Reference chapter: Chapter 4, Section 4.1.1

Answer Explanation
COBS rules on churning and switching require firms to bear the client’s
investment strategy in mind when determining how frequently to deal.

32Where a personal pension plan is sold subject to a cancellation period, within


what MAXIMUM period following conclusion of the contract must a suitability
report be issued?
5 days
10 days
14 days
21 days
Reference chapter: Chapter 4, Section 4.2.2

Answer Explanation
In connection with a personal pension scheme or a stakeholder pension, where
the cancellation rules apply, a suitability report must be provided within 14 days
of concluding the contract.

33A suitability report must be provided to a retail client when an investment


adviser makes a personal recommendation for:

A friendly society policy not exceeding £50 pa


Adding additional contributions to an existing packaged product
An authorised collective investment scheme
An increase to the regular premiums of an existing policy
Reference chapter: Chapter 4, Section 4.2.2

Answer Explanation
Recommending a CIS would require a suitability report while the other options
are examples of exceptions to the requirements.

34What regulatory concession is available to a friendly society that


recommends one of its £30 pa life policies to a retail client?

The risks section of the key features document can be omitted


The data protection wording is not needed on the application form
A reduced cancellation period can be offered
A suitability report is not required
Reference chapter: Chapter 4, Section 4.2.2
Answer Explanation
There are some exceptions to the requirement to provide a suitability report,
including where the personal recommendation is made by a friendly society in
connection with a ‘small life policy’ sold by it, with a premium not exceeding £50
a year or (if payable weekly) £1 a week.

35A firm needs to assess the appropriateness of an investment or service for a


retail client when:

The service is based on shares that trade on a regulated market


The service is only the reception and transmission of the client's orders of a
bond
The service is provided at the initiative of the client
The service is to arrange the purchase of a contingent liability instrument
Reference chapter: Chapter 4, Section 4.3

Answer Explanation
The rules on non-advised sales apply specifically to firms arranging deals, or
dealing, in warrants and derivatives for retail clients.

36An authorised firm is providing execution-only services to retail clients in


relation to warrants. However, the firm has doubts as to the appropriateness of
the deal for a particular retail client. Which of the following best describes how
the firm should proceed?

There is a requirement for appropriateness to be assessed for any execution-


only business
The firm must warn the client before proceeding
The firm must reject the order
The firm can proceed regardless because the transaction relates to warrants
Reference chapter: Chapter 4, Section 4.3.2

Answer Explanation
Firms are required to ask the client for information about their knowledge and
experience of warrants so that they can assess whether the warrant is
appropriate. If a firm believes, based on the above assessment, that the product
or service contemplated is not appropriate for the client, it must warn them of
that fact.

37An authorised firm is providing execution only services to retail clients in


relation to listed shares. However, the firm has doubts as to the appropriateness
of the deal for a particular retail client. Which of the following best describes
how the firm should proceed?

There is no requirement for appropriateness to be assessed for any execution


only business
The firm must warn the client before proceeding
The firm must reject the order
The firm can proceed regardless because the transaction relates to listed
shares
Reference chapter: Chapter 4, Section 4.3.3

Answer Explanation
Firms are not required to ask clients to provide information or assess
appropriateness if the service is execution-only, or for the receipt and
transmission of client orders, in relation to ‘particular financial instruments’
including shares listed on a regulated market or an equivalent third-country
market.

38A key investor information document is required to be produced for which


type of product?

Life policies
Pension products
Protection products
UCITS funds
Reference chapter: Chapter 4, Section 5.1.1
Answer Explanation
A key investor information document (KIID) must be produced for UCITS
schemes; a KFD or illustration would be produced for the other products.

39What is the main purpose of a key investor information document (KIID)?

To enable investors to make well-informed investment decisions


To set out the costs and associated charges of a UCITS scheme
To present clients with a risk/reward profile of the investment
To describe the scheme’s investment policy
Reference chapter: Chapter 4, Section 5.1.1

Answer Explanation
The main purpose of the KIID is to enable investors to reasonably understand
the nature and risks of the investment product being offered to them and,
therefore, make investment decisions on an informed basis.

40A key features document must be provided when:

Selling any product to a retail client


Selling packaged products to a professional client
Selling packaged products to a retail client
Selling packaged products to any client
Reference chapter: Chapter 4, Section 5.1.2

Answer Explanation
A key features document (KFD) must be provided to retail clients when
recommending packaged products.

41When a firm holds itself out to be independent and makes a personal


recommendation to a retail client to buy a packaged product, its scope must be
which one of the following?
The whole of the market
Restricted to other companies within the same group
Restricted to a single product provider
Restricted to a small selection of product providers
Reference chapter: Chapter 4, Section 5.1.3

Answer Explanation
If it holds itself out as being ‘independent’, then its selection must be sufficiently
wide to satisfy the client’s best interest rule and the fair, clear and not
misleading rule. The whole of the market would obviously satisfy this.

42In respect of the sale of which one of the following products must the
cancellation records be retained indefinitely?

Mortgage-linked endowments
Free-standing AVCs
OEICs
Friendly society savings plans
Reference chapter: Chapter 4, Section 5.2

Answer Explanation
The record-keeping retention period for cancellation records is: indefinitely for
pension transfers, pension opt-out or FSAVCs; at least five years in relation to
a life policy, pension contract, personal pension scheme or stakeholder pension
scheme; and at least three years in any other case.

43An authorised firm is entering into advisory, non-distance contracts with retail
customers in relation to purchasing units in its collective investment schemes.
What is the appropriate cancellation period after the contract is concluded?

Whatever the firm decides is reasonable


5 business days
14 calendar days
30 calendar days
Reference chapter: Chapter 4, Section 5.2

Answer Explanation
For firms entering into advised non-distance contracts with consumers, the
cancellation period for non-life/pensions products such as units in collective
investment schemes from the operator of the scheme is 14 calendar days.

44The cancellation period for cash ISAs under the rules in COBS is which of
the following?

7 days
14 days
21 days
30 days
Reference chapter: Chapter 4, Section 5.2

Answer Explanation
The cancellation period for cash ISAs and non-life and pension products is 14
days and for life and pension products it is 30 days.

45Where a firm identifies a conflict of interest which cannot be avoided, what


specific action is it obliged to take?

Submit a completed notification form to the FCA


Make appropriate disclosures to the affected parties
Appoint a nominated person to manage the situation
Suspend all related activities until the matter is resolved
Reference chapter: Chapter 4, Section 6.2

Answer Explanation
Where a firm identifies a conflict of interest which cannot be avoided, it must
make appropriate disclosures to the affected parties.

46Which one of the following rules applies in relation to Chinese walls


arrangements?

Firms are not permitted to undertake designated investment business until


these are in place
Prior approval must be obtained from the FCA before implementation of these
arrangements
Once in place, employees of the firm are permitted to withhold information from
colleagues in other parts of the firm
Once in place, only senior employees are permitted access to sensitive
information
Reference chapter: Chapter 4, Section 6.2.1

Answer Explanation
SYSC requires that where a firm establishes and maintains a Chinese wall, it
must: withhold or not use the information held; and for that purpose, permit its
employees in one part of the business to withhold the information from those
employed in another part of the business.

47Authorised firms are required to take all reasonable steps to identify conflicts
of interest between all of the following except:

Clients and the firm itself


Clients and appointed representatives of the firm
Clients and suppliers to the firm
One client and another
Reference chapter: Chapter 4, Section 6.2.1

Answer Explanation
Firms are required to take all reasonable steps to identify conflicts of interest
between: the firm, including its managers, employees, appointed
representatives/tied agents and parties connected by way of control and a client
of the firm; and one client of the firm and another. There is no requirement in
relation to suppliers to the firm.

48The primary purpose of the Chinese walls rules is to ensure that information
held by a member of a firm is:

Kept as up to date and accurate as possible


Monitored for potential money laundering activities
Not passed on to certain fellow employees
Retained for a minimum period of time
Reference chapter: Chapter 4, Section 6.2.1

Answer Explanation
‘Chinese wall’ is the term given to arrangements made by a firm, such that in
order to manage conflicts of interest, information held by an employee in one
part of the business must be withheld from (or, if this is not possible, at least
not used by) the people with or for whom he/she acts in another part of the
business.

49Which one of the following best describes a conflicts of interest policy?

It means a firm is not tied to any one product


It ensures employees act with an appropriate level of independence
It determines that each counterparty is liable for its own behaviour
It is operated by collective investment schemes to maintain arm's length dealing
Reference chapter: Chapter 4, Section 6.2.2

Answer Explanation
A firm is required to pay due regard to the interests of clients in situations of
potential conflict. A conflicts of interest policy requires the employee to
disregard any material interest or conflict of interest when advising or dealing
for a client.
50In which one of the following circumstances could a firm deal in an
investment when the firm intends to publish a research recommendation about
it?

As soon as the research has been delivered to clients


For discretionary managed client portfolios only
In order to fulfil anticipated client orders only
For execution-only client orders only
Reference chapter: Chapter 4, Section 6.2.3

Answer Explanation
Firms must ensure that there are arrangements such that financial analysts and
other relevant persons who know the likely timing/content of investment
research which is not yet publicly available, or available to clients and which
cannot be inferred from information that is so available, cannot undertake
personal transactions, or trade for others, until the recipient of the investment
research has had a reasonable opportunity to act on it. However, there are
certain exceptions, such as the receipt of an instruction from an execution-only
client, a market maker acting in good faith, etc.

51An investment manager has been offered an inducement for directing its
equity trades through a particular broker. The inducement will provide the
investment manager with portfolio valuation services. Which one of the
following is most accurate?

The inducement is acceptable as long as it is disclosed to the clients


The inducement is only acceptable if the clients are professional clients
The inducement is only acceptable if it is used for the portfolios of all of the
firm’s clients
The inducement is not acceptable
Reference chapter: Chapter 4, Section 6.3.2

Answer Explanation
When an investment manager executes customer orders that relate to certain
designated investments (shares, warrants, certificates representing certain
securities, options and rights to, or interests in, investments of shares), then it
is not permitted to use client-dealing commissions generated from dealing on
behalf of its clients to purchase goods or services, unless these goods or
services relate to execution services or provisions of research. Examples of
goods and services that relate to execution trades or the provision of research
that the FCA does not regard as meeting the requirements include valuation or
portfolio measurement services.

52The best execution rules require a firm to take account of the execution
factors. These specifically include which of the following?

Costs, counterparty, likelihood of execution and size


Counterparty, venue, and soft dollar arrangements
Price, order type, costs and likelihood of settlement
Size, execution venue, order type, and counterparty
Reference chapter: Chapter 4, Section 6.4.1

Answer Explanation
The best execution rules require that firms take all reasonable steps to obtain
the best possible result for their clients, taking into account the execution factors.
These factors are price, costs, speed, likelihood of execution and settlement,
size, nature or any other consideration relevant to the execution of an order.

53What is the minimum frequency at which a firm should review its order
execution policy?

Monthly
Quarterly
Half-yearly
Annually
Reference chapter: Chapter 4, Section 6.4.2

Answer Explanation
Firms must review their order execution policies whenever a material event
occurs, but at least annually.
54A client wants to buy shares. Which one of the following best describes the
way that the firm should execute the order?

Buy the shares at the lowest available price


Buy the shares at the cheapest price available on a regulated market (such as
the LSE)
Buy the shares at least as cheaply as the price displayed on the regulated
market
Buy the shares in a way that gets the best possible result for the client
Reference chapter: Chapter 4, Section 6.4.2

Answer Explanation
Firms are required to establish an order execution policy to enable them to
obtain the best possible results for their clients. This must include, for each
class of financial instrument in which the firm deals, information about the
different execution venues where the firm executes its client orders, and the
factors that will affect the choice of venue used.

55The client order handling rules are designed to prevent an issue with which
of the following?

Conflicts of interest rules


Disclosure rules
Suitability rules
Transparency rules
Reference chapter: Chapter 4, Section 6.5

Answer Explanation
The client order handling rules require comparable client orders to be executed
in the order in which they are received in order to avoid conflicts of interest
where possible.

56If a firm aggregates a client order, any partially executed orders must be
allocated on what basis according to the aggregation and allocation rules?

Depending on the relative size of the order


In a pro rata fashion
In line with the allocation policy
On a first in, first out fashion
Reference chapter: Chapter 4, Section 6.6

Answer Explanation
The rules require that if an aggregated order is only partly executed, the firm
must then allocate the various trades in line with this allocation policy. The
allocation policy may choose one of the other options, but the rules require that
the policy should result in a fair allocation and the approach may therefore differ
from trade to trade.

57Croxteth Investment Bank places an aggregated order embracing both


clients and an own account transaction. The order cannot be satisfied in full.
Presuming that the inclusion of the firm’s order did not bring about more
favourable terms, which one of the following best describes the way the order
should be allocated?

Proportionately to the clients and the firm


To clients first, and only then to the firm
To the firm first, and only then to the clients
In any way that the firm chooses
Reference chapter: Chapter 4, Section 6.6

Answer Explanation
Where a firm has own account deals in an aggregated order along with those
of clients, it must not allocate them in a way which is detrimental to the clients.
In particular, it must allocate the client orders in priority over its own, unless it
can show that, without the inclusion of its own order, less favourable terms
would have been obtained; in these circumstances, it may allocate the deals
proportionately.
58A client has given a firm a limit order that is below normal market size but
which cannot be immediately filled. The firm must make the order public within
what timescale?

Immediately
End of current business day
End of next business day
Within two business days
Reference chapter: Chapter 4, Section 6.7

Answer Explanation
Unless the client instructs otherwise, a firm which receives a client limit order
for shares listed on a regulated market and which it cannot immediately execute
under the prevailing market conditions must make the limit order public
immediately so that it can be executed as soon as possible.

59Mr Barnaby instructs his stockbroking firm to execute an order to buy a small
quantity of ABC plc shares (a listed UK company) at a maximum price of £4 per
share. ABC shares are currently trading at £4.30 per shares. Which one of the
following is most accurate in relation to the FCA rules?

The order must be transmitted to the LSE


The order must be transmitted to either the LSE or another regulated market
The order must be transmitted to the LSE, another regulated market or an MTF
operating an order book trading system
The order can be kept secret by the firm until the price of ABC shares falls to
£4 or less
Reference chapter: Chapter 4, Section 6.7

Answer Explanation
As long as the order is not over the normal market size and the client has not
instructed otherwise, a firm which receives a client limit order for shares listed
on a regulated market and which cannot immediately execute it under the
prevailing market conditions must make the limit order public (in a manner
easily accessible to other market participants) immediately so that it can be
executed as soon as possible. It may do this by: transmitting the order to a
regulated market or MTF operating an order book trading system; or ensuring
the order is made public and can be easily executed as soon as market
conditions allow.

60The personal account dealing rules are intended to prevent which type of
event arising?

Client money breach


Conflicts of interest
Pollution of trust
Suitability breach
Reference chapter: Chapter 4, Section 6.8.1

Answer Explanation
Firms must have arrangements in place to prevent their employees from
entering into a personal transaction that is contrary to the MAD, involves misuse
or improper disclosure of confidential information, or conflicts with the firm’s
duties to a customer.

61Which one of the following is normally required of an employee before


undertaking personal account dealing?

Deal only after informing the firm


Deal only at a specified time of day
Deal only in selected investments
Deal only with their normal broker
Reference chapter: Chapter 4, Section 6.8.2

Answer Explanation
Firms are required to make arrangements and procedures regarding personal
account dealing. These must ensure that the firm is informed promptly of any
transaction, either by notification of it or some other procedure enabling the firm
to identify it.
62When a retail client trade confirmation is received from a third party, within
what time period must the firm pass it on?

On the same day as receipt


No later than the business day following receipt
Within two business days of receipt
Within five business days of receipt
Reference chapter: Chapter 4, Section 7.1

Answer Explanation
For retail clients, firms are required to pass on the details of a trade confirmation
received from a third party no later than the business day following receipt.

63After executing a purchase order, when is a written confirmation normally


expected to be despatched?

Within one business day


Within two business days
Within three business days
Within five business days
Reference chapter: Chapter 4, Section 7.1

Answer Explanation
Where a firm carries out an order for a retail client it must send a notice
confirming the deal details as soon as possible (but no later than on the next
business day). Where the confirmation is received from a third party, the firm
must pass the details on no later than the business day following receipt.

64An investment firm placed an order to buy shares for a client on Monday with
a broker. The trade was executed on Tuesday and the trade confirmation was
sent to the investment firm on Wednesday. At what stage must the client be
provided with a trade confirmation?
Monday
Tuesday
Wednesday
Thursday
Reference chapter: Chapter 4, Section 7.1.1

Answer Explanation
The trade confirmation information should be sent as soon as possible but no
later than on the next business day; if the confirmation is received from a third
party, the firm must pass the details on no later than the business day following
receipt.

65A firm subject to the common platform rules must retain copies of trade
confirmations issued to clients for what period?

Three years from the date of the trade


Three years from the date of dispatch
Five years from the date of the trade
Five years from the date of dispatch
Reference chapter: Chapter 4, Section 7.1.1

Answer Explanation
For MiFID business, firms must keep copies of all confirmations sent to clients
for at least five years from the date of dispatch.

66PYG ltd is an authorised firm that provides brokerage services in large cap
equities. After carrying out an order for a retail client, which one of the following
best describes the transaction reporting requirements?

Send the essential information related to the transaction in a durable medium


within three business days
Send the essential information related to the transaction in a durable medium
within one business day
Convey the essential information orally or in writing on the same business day
Convey the essential information orally or in writing by the end of the next
business day
Reference chapter: Chapter 4, Section 7.1.1

Answer Explanation
Firms must provide the essential information in a durable medium and for retail
clients only, sending a notice confirming the deal details as soon as possible
(but no later than on the next business day).

67A client of a private bank utilises their facility to borrow against their portfolio
to increase the funds available to invest within their investment management
account. How frequently must the firm issue periodic statements to the client?

Monthly
Three-monthly
Six-monthly
Annually
Reference chapter: Chapter 4, Section 7.1.2

Answer Explanation
If the client has authorised that their portfolio be leveraged, the statement must
be provided monthly.

68How frequently must an investment management firm provide a periodic


statement to a retail client who has authorised a geared portfolio?

Monthly
Quarterly
Six-monthly
Annually
Reference chapter: Chapter 4, Section 7.1.2

Answer Explanation
Firms providing investment management services must provide periodic
statements. Where a retail client has authorised that their portfolio be leveraged,
this statement must be provided monthly.

69When must written portfolio statements from investment managers normally


be sent to private customers?

At least every month


At least every three months
At least every six months
After every significant transaction
Reference chapter: Chapter 4, Section 7.1.2

Answer Explanation
Firms providing investment management services must provide periodic
statements, unless these are provided by another party. These must be sent at
least every six months for retail clients.

70A client has authorised that their portfolio may be leveraged. According to
the FCA’s reporting rules, how often must they receive a periodic statement
from an investment manager?

Monthly
Every three months
Every six months
Annually
Reference chapter: Chapter 4, Section 7.1.2

Answer Explanation
Other retail clients must be sent statements every six months, or three months
if this is requested. If the client receives deal-by-deal confirmations, and certain
higher-risk investments are excluded, the statement may be sent every 12
months.
71A firm has paid some of its own money into a client bank account and
subsequently goes bust. What is the consequence of this?

The liquidation cannot proceed until a joint application with the FCA for
directions is made to the court
The liquidator will be able to claim a pro rata share of all of the client money in
order to settle its debts
The liquidator will be able to claim all the money in the client money account
and the clients will become unsecured creditors of the firm
The liquidator will demand the repayment of the firm's money from the account
before anything can be paid to the clients
Reference chapter: Chapter 4, Section 8.1

Answer Explanation
The CASS rules are aimed at ensuring that, if the firm fails, money will not be
used to repay its creditors. When the firm places its own money in the client
money account, this creates a pollution of trust and the liquidator can claim the
whole client money balance and the clients become unsecured creditors.

72A statutory trust is concerned with which of the following?

Client money
Conflicts of interest
Dealing
Suitability
Reference chapter: Chapter 4, Section 8.1

Answer Explanation
This is where a bank account is opened as a client money bank account so that
the money is held in trust for clients.

73Which one of the following is a requirement of the client money rules?

Client money must be reconciled at least once a week


MiFID firms must hold client money with a UK-based bank
Money belonging to clients must be held in a separately designated bank
account
MiFID firms must hold client money on a trustee basis
Reference chapter: Chapter 4, Section 8.1

Answer Explanation
In terms of client money, the main objective is to ensure that it is segregated
from the firm’s own money. Usually this is done by ensuring that it is placed in
a separately designated client money account with a bank and ensuring that
the bank treats it as separate from the firm’s own.

74What is the primary aim of the requirement under CASS to segregate client
money from other money held by the firm?

To prevent money laundering


To prevent client money from being used to pay the firm’s creditors
To satisfy audit requirements
To ensure a speedy distribution of funds should the firm go into liquidation
Reference chapter: Chapter 4, Section 8.1

Answer Explanation
Within CASS, there is a requirement to segregate client money from any other
money held by the firm. This aims to ensure that, if the firm fails, money will not
be used to repay its creditors.

75A reconciliation of a firm's client money records reveals a discrepancy


between the firm's internal records and the client money bank accounts. By
what stage should the excess be withdrawn or any shortfall be paid in while the
reconciliation break is being investigated?

Close of business that day


Next business day
Within 10 business days
Within 25 business days
Reference chapter: Chapter 4, Section 8.2.2

Answer Explanation
If a reconciliation shows a discrepancy, the firm must investigate to identify the
reason for the discrepancy and ensure that either any shortfall is paid into the
client bank account or any excess is withdrawn from the client bank account by
close of business on the day the reconciliation is performed.

76The CASS rules are applied in which scenario?

Coins held for the value of their metal


Discretionary investment management
Money due to the firm
Money held for a DvP transaction
Reference chapter: Chapter 4, Section 8.3

Answer Explanation
The CASS rules are disapplied in certain circumstances such as money due to
the firm, coins held for the value of their metal and money held for a DvP
transaction. They do apply to assets held in a discretionary investment
management account.

77Under which one of the following circumstances would money received from
a client be excluded from the client money rules?

It is reserved by the firm for future fees not yet known


It is accounted for as being owed to the client
It is owed to the firm in the form of fees outstanding
It is paid by cheque to the client but not yet cleared by the bank
Reference chapter: Chapter 4, Section 8.3

Answer Explanation
There are a number of circumstances when the client money rules will not apply,
for example where money becomes due and payable to the firm.

78The FCA’s rules in the Client Assets Sourcebook (CASS) do not normally
apply in which one of the following situations?

An incoming EEA firm holding client money within its passported activities
A firm authorised before 2008
An authorised firm that only holds portfolios for professional clients
An authorised firm holding clients’ investments that are registered overseas
Reference chapter: Chapter 4, Section 8.3

Answer Explanation
Specifically exempt from the CASS requirements are incoming EEA firms, other
than insurers, for their passported activities.
Chapter 1 The Regulatory Environment

1The Senior Managers Regime:


Requires firms regularly to assess the fitness and propriety of individuals
subject to regulatory approval
Replaces the Senior Persons Regime
Replaces the Certification Regime
Is applicable to firms in the UK who deal with customers wherever they are
domiciled
Reference chapter: Chapter 1, Section 5

Answer Explanation
Under the SMR, firms must regularly assess the fitness and propriety of
individuals who are subject to regulatory approval. The Certification Regime
requires relevant firms to assess the fitness and propriety of certain employees
who could pose a risk of significant harm to the firm or any of its customers.
The SMR is applicable to firms in the UK dealing with customers in the UK only.
The SMR supplements the Senior Persons Regime.

2The 'general prohibition' is aimed at which of the following?

Appointed representative
Euroclear UK and Ireland
Independent financial adviser
London Stock Exchange
Reference chapter: Chapter 1, Section 1.1

Answer Explanation
It is a criminal offence to conduct regulated activity by way of business in the
UK unless a person is either authorised to do so, or is an exempt person. An
independent financial adviser requires authorisation and the others are exempt
persons.

3Which one of the following is a statutory operational objective of the FCA?


Supervision of regulated entities
Enforcement of principles
Protecting and enhancing the integrity of the UK financial system
Authorisation of firms and individuals
Reference chapter: Chapter 1, Section 2.1

Answer Explanation
The Financial Services Act 2012 gave the FCA three statutory operational
objectives: the consumer protection objective – securing an appropriate degree
of protection for consumers; the integrity objective – protecting and enhancing
the integrity of the UK financial system; and the competition objective –
promoting effective competition in the interests of consumers in the markets for
regulated financial services or services provided by a recognised investment
exchange in carrying on regulated activities in respect of which it is exempt from
the general prohibition.

4The risks arising from the types of products sold and to whom they are sold
can be classed as which type of risk?

Conduct risk
Liquidity risk
Market risk
Legal risk
Reference chapter: Chapter 1, Section 2.1.4

Answer Explanation
Although there is fundamentally no right answer to the categorisation of conduct
risk, the Financial Stability Board identified business conduct as a new risk
category.

5The Principles for Businesses include which one of the following?

Relations with regulators


Training and Competence
Complaints
Enforcement of Codes
Reference chapter: Chapter 1, Section 3.1

Answer Explanation
Complaints, Training and Competence and Enforcement of Codes are not
headings for Principles for Businesses. Principle 11 covers relations with
regulators.

6The FCA’s requirement for communicating with clients in a way that is clear,
fair and not misleading is set out under which one of the following?

Code of Practice for Approved Persons


Principles for Businesses
Regulatory Powers
Statutory Objectives
Reference chapter: Chapter 1, Section 3.1

Answer Explanation
FCA Principle for Businesses 7 – Communications with clients - requires a firm
to pay due regard to the information needs of its clients and communicate
information to them in a way that is clear, fair and not misleading.

7Undertaking due diligence on a custodian that will hold the assets of


customers’ portfolios is an example of how firms meet which Principle for
Businesses?

Principle 3 – Management and control


Principle 6 – Customers interests
Principle 9 – Customers: relationship of trust
Principle 10 – Client assets
Reference chapter: Chapter 1, Section 3.1
Answer Explanation
Principle 10 states firms must arrange adequate protection for clients’ assets
when it is responsible for them and the CASS rulebook requires them to
undertake due diligence before appointing a custodian and on an ongoing basis
thereafter.

8Which one of the following does the FCA, the FSA’s successor, continue to
define as a consumer outcome in its TCF initiative?

The lack of post-sale barriers to changing products


An imposition of maximum penalty charges on switching
A list of alternative product suppliers
Standard client complaint documentation
Reference chapter: Chapter 1, Section 3.2

Answer Explanation
One of the six defined ‘consumer outcomes’ is that consumers do not face
unreasonable post-sale barriers imposed by firms to change product, switch
provider, submit a claim or make a complaint.

9The FCA's supervision of firms assesses consumer outcomes and where


customers receive advice, that it is suitable and takes into account their
circumstances. To which one of the following does this outcome relate?

Market conduct
Managing conflicts of interest
Protecting client assets
Treating customers fairly
Reference chapter: Chapter 1, Section 3.2

Answer Explanation
The FCA’s Treating Customers Fairly (TCF) initiative includes six ‘consumer
outcomes’. The fourth one states that: "Where consumers receive advice, the
advice is suitable and takes account of their circumstances."

10An employee of a firm would be considered to hold a ‘significant influence


function’ if the role primarily involved:

Giving investment advice


Drafting marketing material
Dealing with complaints
Conducting oversight duties
Reference chapter: Chapter 1, Section 4

Answer Explanation
Significant influence functions cover those functions that are governing or
managerial. They include the directors of the firm and other key personnel.

11Which one of the following is a principle for an approved person?

Manage conflicts of interest fairly using Chinese walls


Observe proper standards of market conduct
Communicate in a clear, fair and not misleading manner
Arrange adequate protection for clients’ assets
Reference chapter: Chapter 1, Section 4.1

Answer Explanation
Principle 3 for approved persons requires the observation of proper standards
of market conduct in carrying out a controlled function; this reflects Principle 5
of the 11 Principles of Business. Clear, fair and not misleading communication,
managing conflicts of interest and protecting clients' assets are Principles for
Businesses.

12Deliberately misleading a client about the risks of an investment is a failure


to comply with which principle for approved persons?
Act with integrity
Act with due skill, care and diligence
Observe standards of market conduct
Deal with regulators in an open way
Reference chapter: Chapter 1, Section 4.2.1

Answer Explanation
The Code of Practice for Statement of Principle 1 - act with integrity - sets out
that deliberately misleading a client about the risks of an investment is a failure
to comply with the requirement for an approved person.

13The establishment of a competent and properly staffed compliance


department is an example under the Code of Practice for which ONE of the
following Statements of Principle?

Principle 1 – act with integrity


Principle 3 – observe proper standards of market conduct
Principle 5 – organisation and control
Principle 7 – complying with relevant requirements and standards of the
regulatory system
Reference chapter: Chapter 1, Section 4.3.3

Answer Explanation
Statement of Principle 7 requires an approved person performing a significant
influence function to take reasonable steps to ensure that the business of the
firm for which he is responsible in his controlled function complies with the
relevant requirements and standards of the regulatory system. This could be
achieved, in part at least, by establishing a competent and properly staffed
compliance department.

14Which one of the following organisations deals with disputes between


consumers and financial services firms?

Financial Conduct Authority


Financial Ombudsman Service
The Office of Fair Trading
Financial Services Compensation Scheme
Reference chapter: Chapter 1, Section 7.1

Answer Explanation
Under FSMA 2000, the Financial Ombudsman Service was established to deal
with disputes between consumers and financial services firms.

15To which organisation can customers submit a claim against an insolvent


financial services firm?

Financial Conduct Authority


Financial Ombudsman Service
Bank of England
Financial Services Compensation Scheme
Reference chapter: Chapter 1, Section 7.2

Answer Explanation
The Financial Services Compensation Scheme provides a safety net for
customers of financial services firms which become unable to repay them when
they become insolvent or default.

16The provisions in the FCA Handbook are generally indicated by the single
letter. Which letter is the provision set out in the FCA Handbook which is a rule
that is not a binding rule in its own right, but relates to another, binding rule?

C
D
E
G
Reference chapter: Chapter 1, Section 8.1
Answer Explanation
Within the FCA Handbook, the type of provision denoted by the letter E
(Evidential Provision) is a rule that is not binding in its own right. It will always
relate to another binding rule. C is for situations that are conclusively not market
abuse, D is for directions and requirements and G is for guidance.

17According to the FCA and the PRA Handbooks, the qualifications tables are
which of the following?

Part of the provisions regarding direction and requirements


An example of an evidential provision
Covered by the Statement of Principle for Approved Persons
Part of the Principles for Businesses
Reference chapter: Chapter 1, Section 8.1

Answer Explanation
Evidential provisions give the required evidence which is expected to show that
a person has complied with, or contravened, a rule. For example, the
qualifications tables have the status of evidential provisions and they relate to
the rule that requires firms to ensure that certain employees attain appropriate
qualifications.

18Acting honestly and fairly at all times when dealing with clients’ interests is
included in which ONE of the following Principles of the Chartered Institute for
Securities & Investment's Code of Conduct?

Principle 1
Principle 2
Principle 5
Principle 7
Reference chapter: Chapter 1, Section 9.1
Answer Explanation
Principle 1 of the CISI's Code of Conduct requires members to act honestly and
fairly at all times when dealing with clients, customers and counterparties and
to be a good steward of their interests.

19The CISI’s published Principles specifically state that members should attain
and actively manage a level of professional competence appropriate to their
responsibilities and also to:

Seek advice from their internal audit department


Promote the development of others
Consult with the firm’s non-executive directors
Approach the firm’s audit committee
Reference chapter: Chapter 1, Section 9.1

Answer Explanation
The CISI's sixth Principle is: To attain and actively manage a level of
professional competence appropriate to your responsibilities, to commit to
continuing learning to ensure the currency of your knowledge, skills and
expertise and to promote the development of others.
Chapter 2 The FSMA 2000 and Financial Services Act 2012

1Under FSMA 2000, what is the maximum penalty for carrying out regulated
activities without being authorised?

Six months' imprisonment and a £5,000 fine


Two years' imprisonment and an unlimited fine
Two years' imprisonment and a £5,000 fine
Five years' imprisonment and an unlimited fine
Reference chapter: Chapter 2, Section 1.1

Answer Explanation
The maximum penalty is two years in prison and an unlimited fine.

2If an adviser carries on specified activities while unauthorised, what can


happen to a contract entered into by a client?

It is void as from day one


It is void as from the date of the conviction of the adviser
It is voidable at the discretion of the adviser
It is voidable at the discretion of the client
Reference chapter: Chapter 2, Section 1.1

Answer Explanation
Any agreement made by a person in contravention of the general prohibition is
unenforceable by that person against the other party. The other party is entitled
to recover any money or property transferred under the agreement and
compensation for any loss suffered.
3What power does the FCA have under S.56 of FSMA 2000?

Prohibit an authorised firm from transacting business


Issue a prohibition order against a person involved in any regulated activity
Issue a warning notice to a firm in breach of a principle
Refer a person in breach of the Conduct of Business rules to the RDC
Reference chapter: Chapter 2, Section 1.1.1

Answer Explanation
Section 56 gives the FCA the power to make a prohibition order against any
person involved in a regulated activity and prohibits that person from performing
specified functions because they are not fit and proper. The prohibition order
may relate only to particular specified regulated activities or may relate to all
regulated activities.

4Which one of the following activities is regulated?

A company establishing an employee share scheme


A company issuing its own securities
A company operating a multilateral trading facility
A company trading in futures to hedge its need for raw materials
Reference chapter: Chapter 2, Section 1.2.2

Answer Explanation
Operating a multilateral trading facility (MTF) is a regulated activity. A company
issuing its own securities, trading in futures to hedge raw materials, or
establishing employee share schemes are all excluded.
5Which one of the following activities is regulated in accordance with FSMA
2000 [Regulated Activities Order]?

Providing Bloomberg share price information


Writing tip sheets
Writing a newspaper column advising on investments
Hosting a financial programme on television
Reference chapter: Chapter 2, Section 1.3.2
Answer Explanation
Newspapers and broadcasters have a specific exemption, as do providers of
factual share price information. However, if the principal purpose of a
publication was the provision of investment advice, ie, tip sheets, then
authorisation would be required.

6Giving investment advice in the money column of a national newspaper is:

Only permitted by an authorised person


An excluded activity under FSMA 2000
Covered by the execution-only selling rules
Subject to compliance sign-off by an authorised firm
Reference chapter: Chapter 2, Section 1.3.2

Answer Explanation
There is a particular exclusion from the regulated activity of ‘advising on
investments’, in relation to newspapers and other media. If a newspaper
includes investment advice, and that advice is not the principal purpose of the
newspaper, then it is excluded from the regulated activity of ‘advising on
investments’.

7What is the regulatory status of a trustee or personal representative carrying


out regulated activities for which they are separately remunerated?

They are required to be authorised


They are exempt
They are excluded from authorisation
They are prohibited
Reference chapter: Chapter 2, Section 1.3.3

Answer Explanation
A trustee or personal representative carrying out regulated activities for which
they are separately remunerated require authorisation. There is an exclusion
under FSMA 2000 from the need for authorisation if the person carrying on a
regulated activity is a trustee or personal representative and not receiving
remuneration for providing investment services.

8Which one of the following is specifically excluded from the need to be


authorised under FSMA 2000?

Employee share schemes


Home purchase plans
Stakeholder pension schemes
Lloyd's underwriting syndicates
Reference chapter: Chapter 2, Section 1.3.4

Answer Explanation
There are a number of categories of exclusion from the need for authorisation,
one of which is employee share schemes. Establishing or running a stakeholder
pension scheme, arranging home purchase plans and managing Lloyd's
underwriting syndicates are all specified activities that require authorisation.

9Which one of the following is specifically excluded from the need to be


authorised under FSMA 2000?

Home reversion plans


Lloyd's underwriting syndicates
Overseas persons
Stakeholder pension schemes
Reference chapter: Chapter 2, Section 1.3.5
Answer Explanation
Overseas persons are one of a number of categories specifically excluded from
the need for authorisation under FSMA 2000.

10The FCA has the power to require information from RCHs under which
section of FSMA 2000?

S.71
S.150
S.165
S.397
Reference chapter: Chapter 2, Section 10

Answer Explanation
Under Section 165 of FSMA 2000, the FCA is given wide-ranging powers to
require information. These powers extend to authorised persons, persons
connected with authorised persons, RCHs and RIEs.

11What power does the FCA have under S.165 of FSMA 2000?

Impose a financial penalty on a firm for a rule breach


Prohibit an authorised firm from transacting business
Require information from any person connected with an authorised person
Vary a firm’s Part 4A permission where it fails to meet a threshold condition
Reference chapter: Chapter 2, Section 10

Answer Explanation
Under Section 165 of FSMA 2000, the FCA is given wide-ranging powers to
require information. These powers extend to authorised persons, persons
connected with authorised persons, RCHs and RIEs.
12What is the purpose of s.89-92 of the Financial Services Act 2012?

To allow the PRA to impose financial penalties for the misconduct of an


approved person
To permit private individuals to sue for damages over a rule breach
To prevent investors being misled or deceived by false actions of others
To provide for the establishment of the Regulatory Decisions Committee
Reference chapter: Chapter 2, Section 11.1

Answer Explanation
The purpose of Sections 89-92 of the Financial Services Act 2012 is to prevent
the actions of investors being driven by reckless, misleading, deceptive or false
actions of others – in essence to protect the integrity of the market.

13What is the name given to the approval granted by the FCA for a firm to
undertake specific regulated activities?

Special authorisation
Part 4A permission
Regulatory exemption
FSMA 2000 permission
Reference chapter: Chapter 2, Section 3.1

Answer Explanation
Part 4A permission is given by the FCA and, once granted, the firm becomes
an authorised person. As an authorised person, the firm can carry on regulated
activities without breaching the general prohibition and committing a criminal
act.
14What is a direct consequence of the regulator giving Part 4A permission?

Authorisation is granted
All threshold conditions must now be assessed
Suitability should now be confirmed
The firm must confirm the application remains complete and accurate
Reference chapter: Chapter 2, Section 3.1

Answer Explanation
When the regulator grants Part 4A permission, the firm becomes an authorised
person and can carry on the specified regulated activities.

15Which FCA supervisory tool is designed to respond to crystallised risks?

Diagnostic
Monitoring
Preventative
Remedial
Reference chapter: Chapter 2, Section 4.1

Answer Explanation
Of the four supervisory tools relating to risk, the remedial tool is designed to
respond to crystallised risks.

16Which of the regulators’ supervisory tools is designed to identify, assess and


measure risk?

Diagnostic
Monitoring
Preventative
Remedial
Reference chapter: Chapter 2, Section 4.1

Answer Explanation
Of the four supervisory tools relating to risk, the diagnostic tool is designed to
identify, assess and measure risk.

17Which of the controlled functions require the person performing it to satisfy


the FCA's fit and proper test?

Significant management function only


Governing function only
All of them except for the customer function
All of them
Reference chapter: Chapter 2, Section 5.2

Answer Explanation
All of the controlled functions require those performing them to satisfy the fit
and proper test.

18Which one of the following criteria for the fit and proper test is a person
disqualified as a director most likely to fail?

Competence and capability


Financial soundness
Honesty, integrity and reputation
Significant influence
Reference chapter: Chapter 2, Section 5.2.1

Answer Explanation
Under the fit and proper test criteria for honesty, integrity and reputation, the
FCA would have regard to whether a person has been disqualified as a director.
19Which category of controlled functions includes the Compliance Officer and
the Money Laundering Reporting Officer (MLRO)?

Required functions
Governing functions
Systems and control functions
Significant management functions
Reference chapter: Chapter 2, Section 5.3

Answer Explanation
Required functions are specific individual functions which the FCA expects
every firm to have, if it is appropriate to the nature of the business. For example,
every firm should have appointed someone to fulfil the money laundering
reporting function and the compliance oversight function.

20Under which one of the following function types does the FCA classify a
director of an unincorporated association?

Governing functions
Required functions
Systems and control functions
Significant management functions
Reference chapter: Chapter 2, Section 5.3

Answer Explanation
Governing functions are the persons responsible for directing the affairs of the
business, for example a director of an unincorporated association.

21Those who provide the information to meet the requirements of Principle 3


(Management and Control) of the Principles for Businesses are classified under
which one of the following functions?

Customer oversight function


Governing control function
Required systemic function
Systems and control function
Reference chapter: Chapter 2, Section 5.3

Answer Explanation
Persons who are classified under the systems and control function are those
who provide the governing body of the firm with the information it needs to meet
the requirements of Principle 3 of the Principles for Businesses.

22Which one of the following is a significant management function?

Head of Settlements
Unit trust salesperson
With-profits actuary
Money laundering reporting officer
Reference chapter: Chapter 2, Section 5.3

Answer Explanation
Significant management functions include those who have responsibility for a
significant business unit, such as Head of Equities or Head of Settlements. A
money laundering reporting officer and with-profits actuary are required
functions, while a unit trust salesperson is a customer function.

23The role of Money Laundering Reporting Officer (MLRO) will normally fall
into which one of the following controlled functions?

Required functions
Systems and control functions
Governing functions
Significant management functions
Reference chapter: Chapter 2, Section 5.3
Answer Explanation
Required functions are specific individual functions which the FCA expects
every firm to have, if it is appropriate to the nature of the business. For example,
every firm should have appointed someone to fulfil the money laundering
reporting function and the compliance oversight function.

24Any employment agreement clause that precludes a person from informing


regulatory authorities of concerns coming to their attention at work would be
void under which one of the following pieces of legislation?

Criminal Justice Act 1993


Data Protection Act 1998
Financial Services and Markets Act 2000
Public Interest Disclosure Act 1998
Reference chapter: Chapter 2, Section 8

Answer Explanation
The Public Interest Disclosure Act 1998 (PIDA) makes any clause or term in an
agreement between a worker and their employer void if it precludes a worker
from whistleblowing.

25Which one of the following Acts provides protection for employees who blow
the whistle?

Financial Services and Markets Act 2000


Public Interest Disclosure Act 1998
Criminal Justice Act 1998
Data Protection Act 1998
Reference chapter: Chapter 2, Section 8

Answer Explanation
The Public Interest Disclosure Act 1998 (PIDA) provides protection for
employees making disclosures, against victimisation by their employers.
26A primary role of the Regulatory Decisions Committee (RDC) is to:

Act as an appeals body for FCA regulatory enforcement


Adjudicate as to whether statutory notices should be issued
Take responsibility for the introduction of disciplinary procedures
Investigate potential cases of wrongdoing on behalf of the FCA
Reference chapter: Chapter 2, Section 9.1

Answer Explanation
The RDC is relatively independent of the FCA, with only its chairman an FCA
employee. As its name suggests, it decides whether to issue statutory notices
and takes the final decisions in areas such as whether or not to discipline and
the appropriate sanction.

27Which one of the following pieces of information is given in a Decision Notice


issued by the FCA?

Decision not to proceed with the action


Details of the effective date
Right of appeal by the recipient
Terms of the final action
Reference chapter: Chapter 2, Section 9.2

Answer Explanation
A Decision Notice gives details of the action that the FCA has decided to take
but leaves room for appeal by the recipient. The effective date and terms of the
final action will come later. A decision not to proceed is a ‘notice of
discontinuance’.
28Which one of the following statutory notices gives the recipient the right to
make representations to the FCA?

Decision Notice
Final Notice
Supervisory Notice
Warning Notice
Reference chapter: Chapter 2, Section 9.2

Answer Explanation
Warning Notices give the recipient details about the action the FCA proposes
to take and the right to make representations to the FCA. Decision Notices give
details of the action that the FCA has decided to take, but leave room for appeal
by the recipient. Final Notices are issued after the opportunity to appeal.
Supervisory Notices give the recipient details regarding the action the FCA
proposes to take or has taken.

29Which type of statutory notice can only be issued with the consent of the
recipient?

Decision Notice
Notice of Discontinuance
Final Notice
Further Decision Notice
Reference chapter: Chapter 2, Section 9.2

Answer Explanation
Further Decision Notices may follow the issue of a Decision Notice where the
FCA has agreed with the recipient to take a different action to that proposed in
the original Decision Notice. The FCA can only issue a Further Decision Notice
with the consent of the recipient.
30What type of notice is normally the first statutory notice giving details of
disciplinary action that the FCA proposes to take?

Decision Notice
Warning Notice
Supervisory Notice
Notice of Discontinuance
Reference chapter: Chapter 2, Section 9.2

Answer Explanation
Once the FCA becomes aware of a problem, a warning notice is issued giving
the recipient details about the action that the FCA proposes to take and the
right to make representations. Once disciplinary action has been decided the
Regulatory Decisions Committee (RDC) would issue a Decision Notice. A
Supervisory Notice is only issued in extreme circumstances and gives the
recipient details regarding the action the FCA has taken, or proposes to take.
A Notice of Discontinuance is where the FCA discontinues its initially planned
disciplinary action.

31Which one of the following measures might the FCA take as remedial action
rather than disciplinary action?

Cancelling the firm’s Part 4A permission


Imposing a financial penalty on the firm
Issuing a public statement of misconduct
Publishing a public censure on the firm
Reference chapter: Chapter 2, Section 9.3

Answer Explanation
The measures available to the FCA to use where it considers it necessary to
take protective or remedial action include: issuing a private warning; varying or
cancelling a firm’s Part 4A permission; withdrawing an individual’s approved
person status; and prohibiting an individual from performing a particular role in
relation to a regulated activity. There are three possible forms of disciplinary
sanction: public statements of misconduct; public censures; and financial
penalties.
32Which one of the following measures would the FCA consider using as
disciplinary action?

Prohibiting an individual from a specified role


Publishing a public censure on a firm
Varying a firm’s Part 4A permission
Withdrawing an individual’s approved person status
Reference chapter: Chapter 2, Section 9.3

Answer Explanation
There are three possible forms of disciplinary sanction: public statements of
misconduct; public censures; and financial penalties. The measures available
to the FCA to use where it considers it necessary to take protective or remedial
action include: issuing a private warning; varying or cancelling a firm’s Part 4A
permission; withdrawing an individual’s approved person status; and prohibiting
an individual from performing a particular role in relation to a regulated activity.

33One of the prescribed situations which could cause the FCA to withdraw a
firm’s Part 4A permission would be where the firm:

Is overdrawn at its bank for more than six months


Employs an individual who has previously failed the fit and proper test
Gives poor advice to a client on more than six occasions
The firm’s regulated activities have come to an end
Reference chapter: Chapter 2, Section 9.3.2

Answer Explanation
The Part 4A permission granted to the firm by the FCA can be withdrawn on
the FCA’s own initiative where the FCA has very serious concerns about the
firm or the way it does business, or where the firm’s regulated activities have
come to an end and it has not applied for cancellation of its Part 4A permission.
34Which section of FSMA 2000 gives the FCA the power to issue a prohibition
order against a person involved in any regulated activity?

S.56
S.59
S.71
S.150
Reference chapter: Chapter 2, Section 9.3.2

Answer Explanation
Section 56 of FSMA gives the FCA the power to make a prohibition order
against any person involved in a regulated activity and prohibits that person
from performing specified functions because they are not fit and proper. The
prohibition order may relate only to particular specified regulated activities, or
to all regulated activities.

35Which one of the following measures is most likely to be used by the FCA
where a firm no longer meets its threshold conditions?

Financial penalty
Public censure
Prohibition
Variation of Part 4A permission
Reference chapter: Chapter 2, Section 9.3.2

Answer Explanation
Where a firm is no longer able to meet the threshold conditions in relation to
one or more of its regulated activities, the FCA would be likely to issue a
variation of Part 4A permission to limit its ability to perform regulated activities
to those areas where it still meets the threshold conditions. A financial penalty,
public censure and prohibition may be applied as well - but this will depend
upon whether any other rule contravention occurred.
Chapter 3 The FCA’s Conduct of Business and Client Assets Sourcebooks

1.Under the Bribery Act 2010 the standard for deciding what is expected
behaviour is:

Customary practice wherever the action took place

What a reasonable person where the action took place might expect

What a reasonable person in the UK might expect

How a reasonable person might interpret common law

Reference chapter: Chapter 3 Section 3.7.2

Answer Explanation
Section 5 of the Bribery Act 2010 provides that the standard in deciding what
would be expected is what a reasonable person in the UK might expect.
Outside the UK, local practice should be disregarded unless codified. The
Bribery Act abolished the old common law provisions.

2
Which one of the following investments is outside the scope of the insider
dealing legislation?

Options on the FTSE 100 Index

Gilts traded on the London Stock Exchange

Commodity futures

Shares traded on AIM


Reference chapter: Chapter 3, Section 1.4
Answer Explanation
For the purpose of the insider dealing legislation securities are: shares; debt
securities (issued by a company or a public sector body); warrants; depositary
receipts; options (to acquire or dispose of securities); futures (to acquire or
dispose of securities); and contracts for differences (based on securities,
interest rates or share indices). However, this definition does not embrace
commodities, and derivatives on commodities (such as options and futures on
agricultural products, metals or energy products).

3
Which one of the following general defences available under insider dealing
legislation relates only to the offence of disclosing insider information?

The individual did not expect to profit

The individual would have dealt anyway

The individual believed the information had been widely disclosed

The individual did not expect anyone to deal on the information

Reference chapter: Chapter 3, Section 1.5

Answer Explanation
There are four general defences available under the insider dealing
legislation. The defence that the individual did not expect anyone to deal on
the information disclosed relates only to the offence of disclosing information
to another person.
4
Which one of the following pieces of legislation gives the FCA the power to
prosecute for insider dealing offences?

Criminal Justice Act 1993

Data Protection Act 1998

Financial Services and Markets Act 2000

Public Interest Disclosure Act 1998

Reference chapter: Chapter 3, Section 1.7

Answer Explanation
FSMA 2000 gives the FCA the power to prosecute for insider dealing
offences.
5
What standards determine if behaviour constitutes market abuse?

Those contained in approved practitioner codes

Those required by each individual market

Those set out in the Upper Tribunal's code

Those expected by a regular user

Reference chapter: Chapter 3, Section 2.1


Answer Explanation
FSMA 2000 sets out that the determination of whether or not behaviour
amounts to market abuse requires consideration of how a hypothetical
reasonable person (the ‘regular user’), familiar with the market in question,
would view the behaviour.

6
An investment manager deals in both calls and puts. Trading in which of
these, if either, is covered by the offence of market abuse under the Financial
Services and Markets Act (FSMA) 2000?

Neither

Calls only

Puts only

Both

Reference chapter: Chapter 3, Section 2.1

Answer Explanation
The offence of market abuse covers investments traded on any of the UK’s
RIEs, investments where application has been made for trading on such
markets and related investments like derivatives. Trading in both calls and
puts is therefore covered by the offence of market abuse under FSMA 2000.
7
A regular user of which one of the following markets would not be covered by
the market abuse legislation?

EDX

LME

NYSE

ISDX

Reference chapter: Chapter 3, Section 2.1

Answer Explanation
The offence of market abuse covers ‘qualifying investments’ on ‘prescribed
markets’ – broadly, they are the investments traded on any of the UK’s
recognised investment exchanges. The NYSE is a designated investment
exchange and is, therefore, not subject to the market abuse legislation.

8
Which investment exchanges are covered by the market abuse provisions of
the Financial Services and Markets Act 2000?

The London Stock Exchange only

The London Stock Exchange and LIFFE only

The UK recognised investment exchanges only

The UK recognised investment exchanges plus NASDAQ and NYSE


Reference chapter: Chapter 3, Section 2.1

Answer Explanation
The market abuse provisions of the Financial Services and Markets Act 2000
cover recognised investment exchanges only.

9
Which one of the following Codes provides guidance on market abuse
offences?

FCA Code of Market Conduct

CISI Code of Conduct

UKLA Model Code

Takeover Code

Reference chapter: Chapter 3, Section 2.2

Answer Explanation
The FCA Code of Market Conduct forms part of the FCA Handbook and
gives guidance on what does and does not constitute market abuse and the
factors taken into account in determining if an offence has been committed.

10
Which one of the following organisations provides guidance notes on market
abuse behaviour?

FCA

HMRC
JMLSG

LSE

Reference chapter: Chapter 3, Section 2.2

Answer Explanation
The FCA’s role in relation to market abuse requires it to prepare and issue a
‘code’ providing guidance to determine whether or not behaviour amounts to
market abuse. This ‘Code of Market Conduct’ forms part of the FCA
Handbook.

11
The offence of effecting transactions by employing fictitious devices falls
under which one of the following?

Insider dealing

Market abuse

Money laundering

Proceeds of crime

Reference chapter: Chapter 3, Section 2.2.1

Answer Explanation
There are seven circumstances listed in the FCA’s Code of Market Conduct
sourcebook relating to behaviour that would constitute market abuse including
effecting transactions by employing the use of fictitious devices or other forms
of contrivance.
12
An order to trade which is effected for illegitimate purposes and which secures
the price of an investment at an abnormal level is an example of:

A manipulating transaction

A manipulating device

Misleading behaviour
Distortion

Reference chapter: Chapter 3, Section 2.2.1

Answer Explanation
A manipulating transaction is defined by the fact that the trade is not for
legitimate reasons. One aim of this can be to secure the price of the
investment at an abnormal level. A manipulating device employs a fictitious
device. Misleading behaviour and distortion are certain actions not covered by
the above.

13
Which one of the following activities is most likely to be classed as market
abuse?

Conducting a transaction after failing to give suitable advice

Carrying out a transaction which breaches money laundering rules

Carrying out a transaction using fictitious financial devices

Conducting excessive transactions in order to generate commission


Reference chapter: Chapter 3, Section 2.2.1

Answer Explanation
Within the ‘Code of Market Conduct’ the FCA explains the types of behaviour
caught by the market abuse regime. They extend to seven circumstances,
including behaviour that consists of effecting transactions or orders to trade
which employ fictitious devices or any other form of deception or contrivance.

14
The concept of effect is used to determine an offence under which one of the
following?

Insider dealing

Market abuse

Money laundering

Proceeds of crime

Reference chapter: Chapter 3, Section 2.3


Answer Explanation
It is not essential for the person responsible for the behaviour in question to
have intended to commit market abuse, ie, market abuse focuses on the
effects of the behaviour, rather than any intention behind the behaviour.

15
Which one of the following pieces of legislation gives the FCA the power to
impose an unlimited civil fine for market abuse?

Criminal Justice Act 1993

Financial Services Act 2012


Financial Services and Markets Act 2000

Public Interest Disclosure Act 1998

Reference chapter: Chapter 3, Section 2.4

Answer Explanation
FSMA 2000 gives the FCA the power to impose a number of penalties for
market abuse, including the imposition of an unlimited civil fine.

16
Where the FCA concludes that a person has encouraged another party to
engage in market abuse, what, if any, is the maximum level of civil fine which
can be imposed?

£5,000

£10,000

£25,000
No maximum

Reference chapter: Chapter 3, Section 2.4

Answer Explanation
Where the FCA concludes that a person has encouraged another party to
engage in market abuse, the penalties available to it include imposing an
unlimited civil fine.
17
The FCA rules on Chinese walls provide a safe harbour against which one of
the following offences?

Failure to report suspicions of terrorism

Insider dealing

Market abuse

Money laundering

Reference chapter: Chapter 3, Section 2.6

Answer Explanation
The FCA rules on Chinese walls provide one of the safe harbours for the
offence of market abuse.

18
Which one of the following activities is a prescribed ‘safe harbour’ against a
charge of market abuse?

Execution-only deals

Multilateral trading

Lloyd's market transactions

Buy-backs

Reference chapter: Chapter 3, Section 2.6


Answer Explanation
There are certain ‘safe harbours’ against a charge of market abuse. These
are: FCA rules; the Takeover Code; and price stabilisation and buy-backs.

19
The launch of the Money Laundering Regulations 2007 was primarily
triggered by:

Requests from banks and building societies

Directives from the EU

Recommendations from SOCA

Joint Money Laundering Steering Group Guidance

Reference chapter: Chapter 3, Section 3.1

Answer Explanation
The Money Laundering Regulations are relatively detailed regulations,
implemented as the result of EU directives, which deal predominantly with the
administrative provisions that firms need to have to combat money laundering.

20
The Serious Organised Crime and Police Act 2005 enables an alleged money
laundering offender to mount a viable defence against a claim of criminal
conduct, purely on the grounds of:

Ill health

Tax efficiency

Overseas legislation
Regulatory ignorance

Reference chapter: Chapter 3, Section 3.1

Answer Explanation
The Serious Organised Crime and Police Act 2005 (SOCPA) amended
certain sections of the Proceeds of Crime Act 2002 (POCA). In particular, one
feature of POCA was that ‘criminal conduct’ was deemed to include anything
which would have been an offence had it been done in the UK, regardless of
where it had actually happened. SOCPA addressed this difficulty – in part at
least – in that there is a defence for alleged offenders if they can show that
they know, or believe on reasonable grounds, that the conduct was not
criminal in the country where it happened.

21
Which one of the following best describes the integration stage of money
laundering?

Origins of the funds are concealed

Cash is physically disposed of

Funds appear legitimate

Proceeds are separated from their source

Reference chapter: Chapter 3, Section 3.2

Answer Explanation
The integration stage of money laundering is where the separation of the
proceeds from the criminally derived source has been successful and the
ultimate beneficiary appears to be holding legitimate funds. The physical
disposal of cash takes place at the first stage – placement. The separation of
the proceeds from their source takes place at the second stage – layering.

22
A criminal is trying to launder money. He sells some shares that were
purchased with the criminally derived money and invests the proceeds in
bonds. This is an example of which stage of the money laundering process?

Integration

Embedding

Placement

Layering

Reference chapter: Chapter 3, Section 3.2

Answer Explanation
Layering involves moving the money around in order to make it difficult for
the authorities to link the placed funds with the ultimate beneficiary of the
money. This might involve buying and selling foreign currencies, shares or
bonds in rapid succession; investing in collective investment schemes or
insurance-based investment products; or moving the money from one country
to another.

23
A client wishes to settle a large transaction with cash rather than by cheque or
BACS. What stage of money laundering might this be an example of?

Integration

Layering

Placement
Positioning

Reference chapter: Chapter 3, Section 3.2

Answer Explanation
The first stage of money laundering is placement, which is the physical
disposal of cash. The second stage is layering, which is the separating of the
proceeds from their source, and the third stage is integration, which is the
provision of apparent legitimacy to criminally derived wealth.

24
Which one of the following best describes the layering stage of money
laundering?

Concealing the origins of the funds

Physical disposal of cash

Provision of apparent legitimacy to the funds

Separation of the proceeds from their source

Reference chapter: Chapter 3, Section 3.2


Answer Explanation
Separation of the proceeds from their source takes place at the second stage
of money laundering – layering. The integration stage is where the separation
of the proceeds from the criminally derived source has been successful and
the ultimate beneficiary appears to be holding legitimate funds. The physical
disposal of cash takes place at the first stage – placement.
25
A client appears to have a lack of concern over penalties imposed when
moving funds from one investment product to another. What stage of money
laundering is this most likely to be an example of?

Integration

Layering

Placement

Positioning

Reference chapter: Chapter 3, Section 3.2


Answer Explanation
Layering involves moving the money around in order to make it difficult for
the authorities to link the placed funds with the ultimate beneficiary of the
money. This might involve buying and selling foreign currencies, shares or
bonds in rapid succession; investing in collective investment schemes or
insurance-based investment products; or moving the money from one country
to another.

26
Depositing criminally obtained cash directly into a building society account is
an example of which stage of the money laundering process?

Integration

Layering

Placement

Phasing

Reference chapter: Chapter 3, Section 3.2


Answer Explanation
Placement is the introduction of the money into the financial system.
Typically, this involves placing the criminally derived cash into a bank or
building society account, a bureau de change or any other type of enterprise
which can accept cash, such as, for example, a casino.

27
Which one of the following sets out the offence of failure to maintain anti-
money laundering records?

Senior Management Arrangements, Systems and Controls Sourcebook

Joint Money Laundering Steering Group Guidance

Money Laundering Regulations 2007

Serious Organised Crime and Police Act 2005

Reference chapter: Chapter 3, Section 3.3.1

Answer Explanation
There are three main requirements of the Money Laundering Regulations
2007. One of these is that firms must carry out certain identification
procedures, implement certain internal reporting procedures for suspicions
and keep records in relation to anti-money laundering activities.

28
Which one of the following pieces of legislation includes the offence of ‘tipping
off’?

Anti-Terrorism Crime Security Act 2001

Money Laundering Regulations 2007


Proceeds of Crime Act 2002

Serious Organised Crime and Police Act 2005

Reference chapter: Chapter 3, Section 3.3.2

Answer Explanation
The Proceeds of Crime Act 2002 establishes five offences. One of these is
‘tipping off’ – giving another person information, knowing or suspecting that a
money laundering report has been made to the authorities, where that
information is likely to prejudice the investigation.
29
Which one of the following is not a condition related to the money laundering
offence of failure to disclose a suspicion?

A person knows or suspects that another person is laundering money

A person acquires the information in the course of a regulated business


activity

A person does not make a report as soon as is practicable

A person facilitates the acquisition of criminal property

Reference chapter: Chapter 3, Section 3.3.2

Answer Explanation
The three conditions that need to be satisfied for the offence of failure to
disclose are: a person knows or suspects that another person is laundering
money; a person acquires the information in the course of a regulated
business activity; and a person does not make the required disclosure as
soon as is practicable. Where a person facilitates the acquisition of criminal
property it falls under the offence of ‘arrangements’
30
Under the Proceeds of Crime Act, what is the penalty for failing to disclose a
suspicion that another person is engaged in money laundering?

A fine and a jail term of up to 14 years

A fine and a jail term of up to 5 years

A fine of up to £500,000 only

A jail term of up to 5 years only

Reference chapter: Chapter 3, Section 3.3.2

Answer Explanation
Failure to disclose is punishable by a fine and a jail term of up to five years.

31
Which one of the following is an example of the money laundering offence of
‘arrangements’?

Acquiring or possessing criminal property

Disguising or transferring criminal property

Facilitating the retention of criminal property by another person

Informing another person that a disclosure has been made

Reference chapter: Chapter 3, Section 3.3.2


Answer Explanation
The offence of arrangements is when a person enters into an arrangement
which the person knows, or suspects, facilitates the acquisition, retention, use
or control of criminal property for another person.

32
An employee of a regulated business received information that made him
suspicious that a breach of the Proceeds of Crime Act had occurred. Why
may he have been not guilty under the ‘failure to disclose’ conditions?

He felt the person concerned was of good character

He held only a junior position

The information was disclosed at a friend’s birthday party

The incident occurred over three months previously

Reference chapter: Chapter 3, Section 3.3.2

Answer Explanation
One of the conditions which need to be satisfied to support ‘failure to
disclose’ is that the information giving rise to the knowledge or suspicion
came to him during the course of business in a regulated sector.

33
Mary has overall responsibility for her firm’s anti-money laundering systems
and controls. To comply with the FCA's rules, this means that she must be:

Aged 30 or over

A qualified accountant or solicitor


A director or senior manager

Under the age of 65

Reference chapter: Chapter 3, Section 3.4.1

Answer Explanation
Each authorised firm must give a director or senior manager (who may also
be the MLRO) overall responsibility for the establishment and maintenance of
effective anti-money laundering systems and controls.

34
Which one of the following sets outs the requirements for enhanced due
diligence on individuals who are considered to be ‘politically exposed
persons'?

Senior Management Arrangements, Systems and Controls Sourcebook

Joint Money Laundering Steering Group Guidance

Money Laundering Regulations 2007

Serious Organised Crime and Police Act 2005

Reference chapter: Chapter 3, Section 3.4.2

Answer Explanation
The latest guidance from JMLSG sets outs the situations where enhanced
‘due diligence’ must be carried out on persons who are considered to be
‘politically exposed persons’.

35
At what stage does the JMLSG Guidance set out when satisfactory
identification evidence is to be obtained for a customer?

As soon as practicable after the first contact

At the first meeting

At the same time as the first transaction

At the time of the first valuation

Reference chapter: Chapter 3, Section 3.4.3

Answer Explanation
The JMLSG Guidance requires that satisfactory identification evidence for the
customer should be obtained, and verified, as soon as is reasonably
practicable after the first contact between the firm and the customer.

36
The use or threat of action resulting in which one of the following is not a
terrorist activity as defined by the Terrorism Act 2000?

Creating serious risk to public health or safety

Disguising the source of investment funds

Disrupting an electronic system

Endangering a person’s life


Reference chapter: Chapter 3, Section 3.6

Answer Explanation
The Terrorism Act 2000 defines what amounts to terrorism as the use or
threat of action where it: involves serious violence against a person or serious
damage to property; endangers a person’s life; creates serious risk to the
health or safety of the public; or is designed to seriously interfere or disrupt an
electronic system. Disguising the source of investment funds is a money
laundering offence.

37
Under the UK Bribery Act 2010 how should senior management make its
commitment to combating bribery known within the organisation?

Applying due diligence on the persons who perform services on its behalf

Communicating its policies in its annual report

Setting severe penalties for breaches of its policies

Training its staff and communicating its policies

Reference chapter: Chapter 3, Section 3.7.4

Answer Explanation
A firm has a defence to the offence of failing to prevent a bribe if it can show
that it has put in place adequate procedures to prevent bribery.
Communicating policies and procedures to staff and to others who will
perform services enhances awareness and helps to deter bribery by making
clear the basis on which the organisation does business.
38
Under the Model Code, directors are prevented from dealing in their
company’s shares during what maximum period prior to publication of the full-
year accounts?

30 days

60 days

90 days

120 days

Reference chapter: Chapter 3, Section 4

Answer Explanation
The Model Code specifies that directors should not deal during the ‘closed
period’ – the two months leading up to the publication of the company’s
annual financial report and the 30 days leading up to the publication of any
quarterly accounts.

39
Which one of the following forms part of the Listing Rules in relation to share
dealing by directors and senior employees?

Conduct of Business rules

Model Code

Sourcebook on Directors and Insider Dealing


Code of Market Conduct

Reference chapter: Chapter 3, Section 4

Answer Explanation
As part of its Listing Rules, the UK Listing Authority (UKLA) has produced the
‘Model Code’. The Model Code on directors’ dealings outlines how directors
and senior employees are expected to behave when dealing in shares of the
company they work for.

40
What is the ‘close period’ in the context of the Model Code for directors in
relation to quarterly results announcements?

10 days

21 days

30 days

90 days

Reference chapter: Chapter 3, Section 4


Answer Explanation
The close period is the 30 days before quarterly results and the 60 days
before the annual results.

41
Which document sets out restrictions on directors’ investments in their own
company’s shares?

Code of Market Conduct


Conduct of Business Rules

Model Code

Sourcebook on Directors and Insider Dealing

Reference chapter: Chapter 3, Section 4

Answer Explanation
The Model Code on directors’ dealings outlines how directors and senior
employees are expected to behave when dealing in shares of the company
they work for.

42
A director is prohibited from dealing in his own company’s shares for one
month prior to the publication of which one of the following?

AGM notice

Full-year accounts

Half-year accounts

Quarterly accounts

Reference chapter: Chapter 3, Section 4

Answer Explanation
The Model Code specifies the closed period as being one month prior to the
publication of any quarterly accounts and two months prior to the publication
of a company’s full-year accounts.
43
Which one of the following is not a purpose of the Disclosure and
Transparency Rules?

Detail the confidentiality requirements to prevent insider dealing

Promote the prompt and fair disclosure of relevant inside information

Set out circumstances where a delay in the public disclosure of information is


permitted

Specify the circumstances for directors’ dealings

Reference chapter: Chapter 3, Section 5

Answer Explanation
The Disclosure and Transparency rules aim to: promote the prompt and fair
disclosure of relevant information to the market; set out circumstances where
an issuer can delay the public disclosure of inside information; and set out the
requirements to ensure inside information is kept confidential in order to
protect investors and prevent insider dealing. The Model Code sets out the
circumstances for directors' dealings.

44
The Disclosure Rules are related to which of the following?

Data protection

Insider dealing

Money laundering
Prudential requirements

Reference chapter: Chapter 3, Section 5

Answer Explanation
The purpose of the Disclosure Rules is to prevent inside information market
abuse.

45
Which set of rules enables an issuer to delay the release of information in
order to prevent market abuse?

Corporate Governance Code

Disclosure Rules

Model Code

EMIR rules

Reference chapter: Chapter 3, Section 5

Answer Explanation
The Disclosure Rules set out specific circumstances in which an issuer can
delay the public disclosure of inside information.
46
Guidance for issuers on the circumstances where a delay in the public
disclosure of inside information is permitted is set out in which one of the
following FCA Sourcebooks?

DTR

COBS

TC

SYSC

Reference chapter: Chapter 3, Section 5

Answer Explanation
The DTR Sourcebook (the Disclosure and Transparency Rules) contains the
provisions to ensure that information relating to publicly listed securities is
properly handled and disseminated.

47
The ‘data controller’ for a small stockbroking firm must be registered with
which body?

Office of Fair Trading

Financial Conduct Authority

Information Commissioner
Financial Skills Partnership

Reference chapter: Chapter 3, Section 6.1


Answer Explanation
Any firm determining the way personal data is held and processed is a ‘data
controller’ and is, therefore, responsible for compliance with the Data
Protection Act; all data controllers must be registered with the Information
Commissioner.

48
Which one of the following monitors data controllers under the Data Protection
Act?

Department for Business, Innovation and Skills

PRA

HM Revenue & Customs

Information Commissioner

Reference chapter: Chapter 3, Section 6.1

Answer Explanation
Any firm determining the way personal data is held and processed is a data
controller and must be registered with the Information Commissioner. The
Information Commissioner has the authority and power to fine firms that do
not comply with its requirements in respect of data security.
49
Under the Data Protection Act, for what maximum period can data be held?

Three years

Five years

Seven years

For whatever period is necessary for its purpose

Reference chapter: Chapter 3, Section 6.1

Answer Explanation
The Data Protection Act lays down eight principles, which must be complied
with. One of these is that personal data shall not be kept for longer than is
necessary for its purpose or purposes.

50
The European Markets Infrastructure Regulation (EMIR) relates to which type
of instrument?

Bonds

Equities

Futures

OTC derivatives

Reference chapter: Chapter 3, Section 8.6


Answer Explanation
The European Markets Infrastructure Regulation (EMIR) relates to the
transparency of the OTC derivatives market.

51
Why, if at all, would a firm report an unexecuted transaction to the Financial
Conduct Authority (FCA)?

It wouldn't, as only executed transactions need to be reported

It is a requirement of the regime for market abuse

It is a requirement of the firm's duty to report suspicious transactions

It would fall under the requirements of the FCA's Principles for Businesses

Reference chapter: Chapter 3, Section 2.5

Answer Explanation
The obligation to report suspicious transactions extends only to those which
are executed; however, Principle 11 of the Principles for Businesses also
requires that firms disclose everything of which the FCA would reasonably
expect notice, and this is interpreted as giving them grounds to report
unexecuted transactions as well.

52
What actions, if any, would the Financial Conduct Authority (FCA) take
against a person who is being prosecuted for insider dealing?

It wouldn't take any action

It would suspend the person's authorisation until the criminal case is


concluded
It would terminate the person's authorisation
It would investigate the situation and impose a fine if the case is proven

Reference chapter: Chapter 3 Section 2.7

Answer Explanation
The FCA has stated that it is its policy not to impose sanctions for market
abuse if a person is being prosecuted for insider dealing.
Chapter 5 Complaints and Redress

1As part of the requirements for business partnerships to be classed as


'eligible complainants' for complaints handling purposes, their annual turnover
must be below what threshold?

€210,000

€500,000

€2 million

€1 million
Reference chapter: Chapter 5, Section 1.3

Answer Explanation
The internal complaints procedures are only required for 'eligible
complainants' which embraces, inter alia, businesses with a group annual
turnover of less than €2 million at the time the complaint is raised.

2Which of the following is not an eligible complainant?

A trustee of a trust with a net value of less than £1 million


A person using the services of an authorised firm
A person considering using the services of an authorised firm
The spouse of a person using the services of an authorised firm
Reference chapter: Chapter 5, Section 1.3

Answer Explanation
In the case of the spouse of a person using the services of an authorised firm,
the individual does not have a business relationship with the firm. The other
three options are all eligible complainants, providing the complaint arises out
of the business relationship with the firm.
3At the appropriate point in the complaints procedure, firms must advise
complainants of their right to refer the matter to the:

Financial Action Task Force


Financial Services Compensation Scheme
Office of Fair Trading
Financial Ombudsman Service
Reference chapter: Chapter 5, Section 1.4

Answer Explanation
Internal complaints handling procedures should provide for: the receiving of
complaints; responding to complaints; appropriately investigating complaints;
and notifying complainants of their right to go to the Financial Ombudsman
Service (FOS) where relevant.

4If a customer has an unresolved complaint against an authorised firm, who


will deal with it?

Financial Services Compensation Scheme


Office of Fair Trading
Financial Ombudsman Service
Financial Action Task Force
Reference chapter: Chapter 5, Section 2.1

Answer Explanation
The Financial Ombudsman Service (FOS) is used by customers where the
firm has been unable to resolve a complaint to their satisfaction.

5The Financial Ombudsman Service is:

A department within the Financial Conduct Authority


A part of the Office of Fair Trading
An independent body
An organisation formed by member firms
Reference chapter: Chapter 5, Section 2.1

Answer Explanation
Under the provisions of FSMA, the FCA has the power to make rules relating
to the handling of complaints, and an independent body, the Financial
Ombudsman Service, has been established to administer and operate a
dispute resolution scheme.

6An authorised firm has become a participant to a complaint that will be


considered by the Financial Ombudsman Service (FOS). Which one of the
following is true of the binding award that can be made against the firm?

The FOS can award a monetary amount of up to £100,000 plus additional


compensation for pain, suffering and distress
The FOS can award a monetary amount of up to £150,000 plus additional
compensation for pain, suffering and distress
The FOS can award a monetary amount of up to £100,000 inclusive of any
compensation for pain, suffering and distress
The FOS can award a monetary amount of up to £150,000 inclusive of any
compensation for pain, suffering and distress
Reference chapter: Chapter 5, Section 2.3

Answer Explanation
Since January 2012 the FOS’s binding award has been increased from
£100,000 to £150,000. The FOS’s award includes any compensation for
financial loss, pain and suffering, damage to reputation and distress or
inconvenience.
7Under the Financial Services Act 2012, who may make a super-complaint?

Financial Ombudsman Service


A firm itself
Certain consumer bodies
A group of more than ten individuals
Reference chapter: Chapter 5, Section 4
Answer Explanation
Under the Financial Services Act 2012, certain consumer bodies are allowed
to make super-complaints where a feature of the UK financial services market
is, or appears to be, significantly damaging to the interests of consumers. The
FOS and firms themselves are allowed to make mass-detriment references.

You might also like